This is a weakening question: Which one of the following, if true, most seriously undermines the conclusion above?
Our stimulus begins by telling us that surgery was restricted to emergencies only following a relatively high mortality rate in area hospitals. Sounds like a smart decision! It seems the plan worked, and deaths fell by almost a third during the period of restriction. Unfortunately however, when non-emergency surgeries were allowed to resume, the death rate rose again. From all of this the argument concludes that the risks of elective (i.e non-emergency) surgery had been often unnecessarily incurred in the area. In other words, people were risking their lives in surgeries that they didn’t really need. We should notice that although our support does give us good evidence that people were dying, we haven’t established whether or not these risks were necessary. We are assuming that just because a surgery wasn’t an emergency, well then it wasn’t a necessary risk. Our job on this question is to weaken this conclusion that the deaths were due to unnecessary surgeries. Let’s see what our answers have to offer:
Correct Answer Choice (A) This weakens the conclusion by assigning a reason for the elective surgeries which would make possible mortality a necessary risk even if the surgery wasn’t an outright emergency. Sure these weren’t car accident victims who were going to immediately die without surgical intervention, but every day they delayed the surgery their chances of surviving just got worse; it was necessary to perform the surgeries sooner rather than later.
Answer Choice (B) The conclusion is that the risks were incurred unnecessarily, not unknowingly.
Answer Choice (C) This would if anything strengthen the argument by suggesting the surgeries were being unnecessarily performed.
Answer Choice (D) It is entirely consistent for elective surgeries to be, in general, less risky than emergency surgeries, and for hospitals in the area to be incurring unnecessary risks performing said surgeries.
Answer Choice (E) This doesn’t address the argument, it just distinguishes surgery failure from surgery mortality.
This is a weakening question, indicated by: Which one of the following, if true, undermines the conclusion concerning words for colors?
The stimulus begins with the fact that many languages have distinct words for the brother of your mother and the brother of your father, while in English both are referred to as “uncle”. The next sentence concludes that this is evidence of a more finely discriminated kinship system than that of English speakers. Basically, those cultures care more about the minutia of family than English-speaking cultures. We then get another premise, informing us that basic words for colors vary between languages, followed by another conclusion that speakers of languages with fewer words for color must be unable to distinguish as many colors as speakers of English. What a bad argument! The reasoning however is pretty clear; both arguments infer that a difference in language reflects a difference in reality. We’re specifically told which of the conclusion we are to weaken in the question stem, so we should look for an answer choice which would make the color conclusion significantly less likely to be true. Let’s see our options:
Correct Answer Choice (A) This gives us a case where one group is able to distinguish colors but only has one word, directly weakening the argument’s reasoning that number of words and distinguished colors are one to one.
Answer Choice (B) This is completely consistent with the conclusion being true.
Answer Choice (C) This seems to suggest they do distinguish colors differently from English speakers; while unripe bananas are green, I haven’t seen many blue leaves!
Answer Choice (D) This adds nothing.
Answer Choice (E) This is completely consistent with the conclusion being true.
We should recognize this as a resolve, reconcile, explain question, since it asks: Which one of the following, if true, most helps to explain how the deer mouse might have found its way back to its nest?
Our first sentence informs us that deer mice don’t stray far from their nests, and further specifies that when they are moved more than half a kilometer from the nest, they generally never find their way back. Interesting! Since we read the question stem first, we know the discrepancy we are tasked with explaining is that despite the truth of this general statement about the difficulty of deer mice finding their way home, one deer mouse was able to. And that’s exactly what the next sentence introduces, starting with yet which indicates the shift towards our phenomenon in need of an explanation. Our phenomenon is that a young deer mouse was moved two kilometers away, which is four times the distance where a deer mouse will usually never find its way home, and yet was able to return in less than two days. An answer choices which explains how the young deer mouse did this in a way that is consistent with deer mice generally struggling to find their way home will be the correct answer. Let’s see our options:
Answer Choice (A) We’ve been told nothing about how the environment affects a deer mouse’s ability to navigate, or really anything that would make the difference between the deer mouse’s home and where it is moved matter except for distance.
Correct Answer Choice (B) The researchers camped next to the deer mouse nest for several weeks, so it would make sense if the mouse recognized the campfire smell! The answer even specifies that the smoke drifted to the area the deer mouse was released. What this answer does is make the deer mouse’s situation exceptional, such that it is no longer weird that most of the time deer mice will fail to find their way home. Most of the time they don’t have a strong recognizable scent carrying kilometers away from their home!
Answer Choice (C) We’ve been told nothing about how the number deer mice affects a deer mouse’s ability to navigate.
Answer Choice (D) That was nice of them, but it doesn’t do anything for us! If anything this answer makes the situation more confusing, as the deer mouse was unable to see where it was taken; it was within a small dark box.
Answer Choice (E) Ok, but how would that explain the deer mouse’s ability to find its way home? This just makes the fact the deer mouse survived and was able to get home even more incredible, without doing anything to explain why this deer mouse was able to break the general trend of deer mouse navigation failure.
This is a weakening question, though I’d forgive you if you failed to immediately identify it as it throws a particularly dense question stem at us. This is one of those question stems that introduces a final premise which is to be added to the stimulus in our judgment of the answer choices, but in this case, it does so in an incredibly confusing way. Our objective is to specifically weaken an interpretation that holds that the lines serve unrelated purposes rather than refer to astronomical phenomena. Despite the density of this question stem, what should tip us off that it is specifically a weakening question is that it asks: which one of the following, if true, most effectively counters.
The first sentence of the stimulus is very strangely worded, but what we should pick up on is that these lines are ancient, about as wide as a footpath, and stretch long distances; sounds like some old roads! We next learn that these lines form giant shapes, and that one where a bunch of lines emerge from a point intersects with one shaped like a bird; ok not sounding so much like roads anymore. The rest of the stimulus concerns the interpretation of an investigator who thinks the markings are landing strips for aliens and argues that they couldn’t have been roads. From him, we learn a bunch of information that casts doubt on the roads hypothesis; the lines run in strange patterns and will sometimes just end in the middle of the desert. Our job is to weaken an explanation of these weird lines which interprets their shapes as serving unrelated purposes, and specifically to do so from a perspective that believes the lines have an astronomical purpose. Let’s see what we get in the answer choices:
Answer Choice (A) Interesting, but we are trying to weaken the interpretation that these specific lines in Peru had no purpose; what North American peoples do isn’t of interest to us.
Correct Answer Choice (B) This answer strongly suggests that the lines have an astronomical purpose which unites the straight lines (observation spot) with the figure (represents a constellation).
Answer Choice (C) This fails to weaken the interpretation that the figures are unrelated; in fact, it seems to suggest the straight lines are part of a complex of patterns and have nothing to do with the bird.
Answer Choice (D) Similar to A, this brings in a different phenomenon and does nothing to explain how the straight lines and bird figure could have together served an astronomical purpose.
Answer Choice (E) This strengthens the hypothesis we want to weaken; if one was made well before the other, it’s far more likely they were unrelated.
This is an EXCEPT resolve, reconcile, explain question, indicated in the question stem by: Each of the following, if true, helps resolve the apparent discrepancy EXCEPT:
The stimulus tells us that on average people spend significantly less time reading now than 50 years ago, but many more books are bought overall. There is an intuitive discrepancy between a decrease in reading and increase in books sold, but we should recognize that there are a lot of ways of explaining it. For example, we are comparing an average to a total; if the population has increased, then we would expect more books to be sold even if the rate at which people read them, and hence bought them, decreased. That also leads to another obvious issue; can we assume people buying books means they read them? If you are anything like me, you are much better at buying books than reading them. Maybe people just have more income to spend so they are buying more books, even though they don’t have as much time to read! Since this is an EXCEPT question, we are looking for the one answer that doesn’t explain the apparent discrepancy. Let’s see what we get!
Answer Choice (A) This is one issue we identified in the stimulus; more people means more books sold, even if, on average, people read less.
Answer Choice (B) Ok, so maybe people are reading less but a larger share of readers purchase their books rather than borrow them. Makes sense!
Answer Choice (C) So a subset of literate people have a reason to purchase more books even if the set as a whole is reading less.
Correct Answer Choice (D) What people do with their books doesn’t really help us if it doesn’t explain how sales could be higher while reading declined
Answer Choice (E) If books are shorter, then people could be reading less while buying more books!
This is a weakening question, as the question stem asks: Which one of the following, if true, casts the most doubt on the accuracy of the above conclusion?
This stimulus is placed in quotes without an identified speaker, but this doesn’t really matter beyond allowing some of the referential phrasing such as this company. It begins with the context that the speaker’s company will not be training more pilots, as they have a waitlist of 400 trained pilots. Alright, that kind of makes sense. This decision is then supported by an argument.
The argument begins with the premise that the five other major companies have roughly ~ 400 pilots as well. The stimulus continues with another premise with the support indicator since, stating that each company is going to only need roughly ~100 pilots. From these two premises about the company waitlists and personnel needs, the author concludes that there will be no shortage of personnel. The assumption this argument depends on is that the individuals waitlisted for each company are distinct. Since each company requires about ~100 people and there are 5 companies, around ~500 pilots are needed by the major companies. For the argument to conclude that there are enough trained pilots to avoid a shortage, it must assume that it isn’t the same 400 pilots waitlisted for each company, as in that case there would actually already be a shortage brewing since there are ~100 less pilots than needed. An answer which picks up on this assumption and contradicts it will be an excellent weakening answer. Let’s see what we get:
Correct Answer Choice (A) This does exactly what we identified with our pre-phrase, it directly contradicts the arguments assumption that there isn’t significant overlap between the waitlists.
Answer Choice (B) Our author’s argument is specifically about the forseeable future, the argument’s conclusion is not about whether or not training will be needed in the long run.
Answer Choice (C) Ok? Our author’s conclusion is a prediction that there will be no shortage in the foreseeable future; whether or not there will be an age gap does nothing to weaken the argument.
Answer Choice (D) If anything this strengthens the argument by supporting the accuracy of the author’s claims.
Answer Choice (E) If other companies are training more pilots, that strengthens the author’s prediction that there won’t be a shortage.